You are on page 1of 29

capital-intensive goods.

PART 1 B
International Business Environment [5] Source: CMA 1281 1-16
94 Questions In most recent years, the U.S. balance of payments has
registered a deficit. This balance of payments deficit is a
measure of the excess of
[1] Source: CMA 0676 1-34
Which of the following economic policies would not tend to A. Exports over imports.
correct a balance of payments deficit in the U.S.?
B. Imports over exports.
A. Increase productivity in the manufacturing of U.S.
exports. C. Imports, private capital outflows, grants, and
remittances over exports and private capital inflows.
B. More effective use of monetary and fiscal policies
to reduce inflation. D. Goods imports over services imports.

C. A redistribution of economic aid and mutual


defense burdens toward western European countries. [6] Source: CMA 0682 1-12
Of the following transactions, the one that would result in a
D. Increase value of U.S. currency in relation to debit entry in the U.S. balance of payments account is the
foreign currencies.
A. Receipt of dividends by an American corporation
from its German subsidiary.
[2] Source: CMA 0680 1-17
The value of the U.S. dollar in relation to other foreign B. Buying of IBM shares by a Kuwaiti investor.
currencies is
C. U.S. export of military equipment to Saudi Arabia.
A. Determined directly by the price of gold because
the value of the U.S. dollar is tied to the price of gold. D. Expenditure of a U.S. resident vacationing in
France.
B. Set by the U.S. government in consultation with
other foreign governments.
[7] Source: CMA 0682 1-13
C. Set along with the value of other currencies held Which one of the following transactions would result in a
by the International Monetary Fund. credit entry in the U.S. balance of payments account?

D. Determined by the forces of supply and demand A. A New York bank pays $5,000 in interest to
on the foreign exchange markets. foreigners.

B. Volkswagen's U.S. subsidiary remits a dividend of


[3] Source: CMA 0680 1-20 $1 million to its parent company in Germany.
When the U.S. dollar is expected to rise in value against
foreign currencies, a U.S. company with foreign currency C. A U.S. exporter buys marine insurance from a
denominated receivables and payables should British insurance company.

A. Slow down collections and speed up payments. D. An Iowa farmer exports grain to Turkey.

B. Slow down collections and slow down payments.


[8] Source: CMA 1282 1-12
C. Speed up collections and speed up payments. One may characterize the current international monetary
system developed by the industrialized countries as a
D. Speed up collections and slow down payments.
A. Clean float. Freely floating exchange rates are
determined solely by the forces of demand and
[4] Source: CMA 1281 1-12 supply.
If a country has only two factors of production, labor and
capital, and it has a relative abundance of capital, the B. Managed or dirty float. Central banks intervene in
country will tend to the foreign exchange market to influence the
exchange rates.
A. Import capital-intensive goods and export
labor-intensive goods. C. Stable-rate system.

B. Refrain from trading owing to the abundance of D. Gold-based system.


capital.

C. Export capital-intensive goods and import [9] Source: CMA 1282 1-13
labor-intensive goods. An overvalued currency can be considered as

D. Place an embargo on the export of A. A tax on exports and a subsidy to imports.

1
investments.
B. A tax on imports and a subsidy to exports.
B. Greater availability of goods and services in
C. A tax on both exports and imports. domestic markets.

D. A subsidy to both exports and imports.


C. Enlarged revenues accruing to its national
government from the imposition of import duties.
[10] Source: CMA 1282 1-14
Given a spot rate of $1.8655 and a 90-day forward rate of D. Increased profits and wages earned by firms and
$1.8723, the pound sterling in the forward market is workers, respectively, in export industries.

A. Being quoted at a premium.


[15] Source: CMA 1285 1-26
B. Being quoted at a discount. The appropriate remedy for the dumping of products by a
foreign firm in the U.S. market would be to
C. Undervalued.
A. Pass "buy American" laws.
D. Overvalued.
B. Impose restrictions on U.S. exports to the
offending country.
[11] Source: CMA 1282 1-17
Disregarding demand for its factors of production, a C. Impose countervailing duties.
country's comparative advantage will lie in those goods
whose production requires comparatively large amounts of D. Deny "most favored nation" treatment to exporters
its of the offending country.

A. Relatively scarce resources.


[16] Source: CMA 1285 1-27
B. Relatively abundant resources. Trade restrictions such as tariffs and import quotas
represent
C. Natural resources.
A. An attempt by the government to bring about a
D. Capital. more equitable distribution of income.

B. An increase in the unit costs of domestic


[12] Source: CMA 1282 1-18 producers who compete with foreign firms.
The difference between tariffs and quotas is
C. A subsidy paid by domestic consumers to foreign
A. That the tariff is expressed as a percentage of producers of the duty-burdened commodities.
price and the quota is expressed as an amount per
unit. D. A subsidy paid by domestic consumers to
domestic producers of the duty-burdened
B. That a tariff limits price and a quota limits commodities.
quantities.

C. That a tariff is a tax and a quota is a subsidy. [17] Source: CMA 1285 1-28
The creation of a regional economic bloc of trading nations
D. That a tariff is a duty, whereas a quota is a such as the European Union (EU),
limitation on quantities.
A. Discourages foreign investment by nonmember
multinational companies.
[13] Source: CMA 1282 1-19
A voluntary export quota is B. Encourages trade between the member nations
and nonmember nations.
A. A form of unilateral import quota.
C. Requires the adoption of a common monetary
B. Meaningless because of its voluntary nature. unit.

C. A form of import quota negotiated with a country. D. Discriminates economically against nonmember
nations.
D. A violation of the World Trade Organization.

[18] Source: CMA 1285 1-30


[14] Source: CMA 1285 1-25 The dominant reason countries devalue their currencies is
The most significant advantage gained by a nation that to
participates in international trade is the
A. Improve the balance of payments.
A. Higher rates of dividend and interest income
received by its citizens and firms from their foreign B. Discourage exports without having to impose

2
controls.
A. Promote multilateral tariff reductions among
C. Curb inflation by increasing imports. nations.

D. Slow what is regarded as too rapid an B. Provide long-term loans to member countries that
accumulation of international reserves. want to improve their infrastructure.

C. Provide a means of financing food imports so that


[19] Source: CMA 1285 1-31 more domestic resources can be shifted to
The U.S. balance of trade is decreased by industrialization.

A. Foreign investments in the United States. D. Provide short-term loans to member nations
experiencing temporary balance of payments
B. U.S. investments in foreign countries. difficulties.

C. U.S. exports.
[24] Source: CMA 0686 1-23
D. U.S. imports. The economic reasoning dictating that each nation
specialize in the production of goods that it produces
relatively more efficiently than other nations and import
[20] Source: CMA 1285 1-32 those goods that are produced relatively more efficiently by
Debt-servicing problems of less developed countries that other nations is called the doctrine of
primarily sell raw materials to the United States would be
eased by A. Efficient trade.

A. A recession in the United States with declines in B. Diminishing returns.


interest rates.
C. Relative competition.
B. An expanding U.S. economy with stable money
supply growth. D. Comparative advantage.

C. An expansion of the lending authority of the World


Bank. [25] Source: CMA 1286 1-16
The balance of trade is the
D. A significant increase in the level of U.S. tariffs.
A. Same as the balance of the current account.

[21] Source: CMA 1285 1-33 B. Balance on the capital account.


The purchasing-power parity exchange rate
C. Balance on all international transactions.
A. Is a fixed (pegged) exchange rate.
D. Balance on the goods transactions in the current
B. Is always equal to the market exchange rate. account.

C. Results in an undervalued currency of countries


that are net importers. [26] Source: CMA 1286 1-17
Special drawing rights (SDRs) are created by the
D. Holds constant the relative price levels in two International Monetary Fund (IMF) and
countries when measured in a common currency.
A. Are based on the value of a basket of five
currencies and pegged to the value of gold.
[22] Source: CMA 1285 1-34
An American importer of English clothing has contracted to B. Are based on the value of a basket of five
pay an amount fixed in British pounds three months from currencies.
now. If the importer worries that the U.S. dollar may
depreciate sharply against the British pound in the interim, it C. Are pegged to the price of gold.
would be well advised to
D. Are a circulating currency like the European
A. Buy pounds in the forward exchange market. currency unit.

B. Sell pounds in the forward exchange market.


[27] Source: CMA 1286 1-18
C. Buy dollars in the futures market. If the annual U.S. inflation rate is expected to be 5% while
the Italian lira is expected to depreciate against the U.S.
D. Sell dollars in the futures market. dollar by 10%, an Italian firm importing from its U.S.
parent can expect its lira costs for these imports to

[23] Source: CMA 1285 1-35 A. Decrease by about 10%.


The basic objective of the International Monetary Fund is
to B. Decrease by about 5%.

3
C. Increase by about 5%. A. The pound sterling is at a discount against the
dollar and undervalued in the forward market.
D. Increase by about 16.6%.
B. The pound sterling is at a premium against the
dollar and overvalued in the forward market.
[28] Source: CMA 1286 1-19
An overvalued foreign currency exchange rate C. The forward pound sterling is at a discount against
the dollar.
A. Represents a tax on exports and a subsidy to
D. The forward pound sterling is at a premium against
imports. the dollar.

B. Represents a subsidy to exports and a tax on


imports. [32] Source: CMA 0687 1-21
The World Trade Organization
C. Has an effect on capital flows but no effect on
trade flows. A. Introduced fixed exchange rates among the United
States, Canada, and members of the European
D. Has no effect on capital flows but does affect Union.
trade flows.
B. Created the International Monetary Fund.

[Fact Pattern #1] C. Encourages reductions in trade barriers between


Suppose that 2 worker-hours are required to produce a countries.
clock radio in Japan, while 3 are required to do the same in
Germany. In addition, 4 worker-hours are required to D. Introduced exchange rates that adjust in response
produce a television in Japan, while 5 are required to do to changes in trade deficits and surpluses.
the same in Germany.

[29] Source: CMA 1286 1-21 [33] Source: CMA 1287 1-24
(Refers to Fact Pattern #1) In the modern world economy, balance-of-payments
Under conditions appropriate for free trade, both Japan deficits and surpluses can be eliminated
and Germany would be better off if
A. Through the market mechanism of flexible
A. Japan produces only televisions while Germany exchange rates.
produces only clock radios and both use trade to
meet the needs for the item not produced locally. B. If all nations adopt tight monetary policies.

B. Japan produces only clock radios while Germany C. Only if trade between nations is curtailed.
produces only televisions and both use trade to meet
the needs of the item not produced locally. D. When the opportunity costs of production are
made the same in all countries.
C. Both produce some of each item and use trade to
meet additional needs of a product.
[34] Source: CMA 1287 1-25
D. Both Japan and Germany produce both products One of the major consequences of international trade
for internal use only with no need for international between nations is
trade.
A. Higher prices for consumers.

[30] Source: CMA 1286 1-22 B. A decreased variety of consumer products.


(Refers to Fact Pattern #1)
In the case of trade between Japan and Germany, the price C. The possibility for total world output to increase.
of clock radios relative to the price of televisions is likely to
be one clock radio for D. Reduced competition for businesses.

A. 2.00 televisions.
[35] Source: CMA 1287 1-26
B. 1.67 televisions. Which one of the following statements concerning
international trade and protection is true?
C. 1.20 televisions.
A. Protection is necessary in order to keep U.S.
D. 0.50 televisions. money in the United States.

B. When two nations trade, one must gain while the


[31] Source: CMA 1286 1-20 other must lose.
Given a spot exchange rate for the U.S. dollar against the
pound sterling of 1.4925 and a 90-day forward rate of C. The United States cannot compete with nations
1.4775 whose labor costs are lower.

4
today is the
D. U.S. imports raise living standards in the United
States. A. United States dollar.

B. Euro.
[36] Source: CMA 1287 1-27
One consequence of the imposition of tariffs or quotas on C. Japanese yen.
imported products is
D. Swiss franc.
A. Lower prices for domestic products that compete
with affected imports.
[41] Source: CMA 0688 1-22
B. Domestic industry opposition to protection from Exchange rates are determined by
imports.
A. Each industrial country's government.
C. Additional consumption of the affected imported
products. B. The International Monetary Fund.

D. Higher prices for the affected imported products. C. Supply and demand in the foreign currency
market.

[37] Source: CMA 1287 1-28 D. Exporters and importers of manufactured goods.
If the value of the U.S. dollar in foreign currency markets
changes from $1 = 6 marks to $1 = 4 marks,
[42] Source: CMA 0688 1-23
A. The German mark has depreciated against the If risk is purposely undertaken in the foreign currency
dollar. market, the investor in foreign currency then becomes

B. German imported products in the U.S. will A. A speculator.


become more expensive.
B. An arbitrageur.
C. U.S. tourists in Germany will find their dollars will
buy more German products. C. Involved in hedging.

D. U.S. exports to Germany should decrease. D. An exporter.

[38] Source: CMA 1287 1-29 [43] Source: CMA 0688 1-24
If consumers in Japan decide they would like to increase Special drawing rights
their purchases of consumer products made in the United
States, in foreign currency markets there will be a tendency A. Were created to reduce world inflation.
for
B. Represent the creation of new international
A. The supply of dollars to increase. liquidity by the International Monetary Fund.

B. The supply of dollars to decrease. C. Are a new currency used by commercial banks in
the international market.
C. The Japanese yen to appreciate relative to the
U.S. dollar. D. Are allocated to less developed countries every
year by the International Monetary Fund.
D. The demand for dollars to increase.

[44] Source: CMA 0688 1-25


[39] Source: CMA 1287 1-30 In foreign currency markets, the phrase "managed float"
If the U.S. dollar declines in value relative to the currencies refers to the
of many of the U.S. trading partners, the likely result is that
A. Tendency for most currencies to depreciate in
A. Foreign currencies will depreciate against the value.
dollar.
B. Discretionary buying and selling of currencies by
B. The U.S. balance of payments deficit will become central banks.
worse.
C. Necessity of maintaining a highly liquid asset, such
C. U.S. exports will tend to increase. as gold, to conduct international trade.

D. U.S. imports will tend to increase. D. Fact that actual exchange rates are set by private
business people in trading nations.

[40] Source: CMA 0688 1-21


The most widely used currency in international business [45] Source: CMA 0688 1-29

5
In trade discussions between the United States and
Japanese governments, if Japan voluntarily agrees to C. The United States will export chips to Taiwan.
restrict automobile exports, which one of the following is
true? D. Taiwan will specialize in the production of chips.

A. This restriction has no effect on the price paid by


the consumer. [49] Source: CMA 1288 1-14
(Refers to Fact Pattern #2)
B. The United States government gains from these Assuming free trade between the United States and
restrictions. Taiwan, the relative prices of soybeans and chips will be

C. Profit margins for Japanese auto manufacturers A. Exactly 1 ton of soybeans for 250 chips.
have increased.
B. Between 150 to 250 chips for 1 ton of soybeans.
D. Under this agreement, the Japanese have an
incentive to export less expensive cars to the United C. Between 1.2 to 2.0 tons of soybeans for 100
States. chips.

D. Exactly 1 ton of soybeans for 120 chips.


[46] Source: CMA 0688 1-30
What is the role of gold in the present international
monetary system? [50] Source: CMA 1288 1-15
The U.S. dollar has a free-floating exchange rate. When the
A. Gold is quoted in United States dollars only. dollar has fallen considerably in relation to other currencies,
the
B. All of the major currencies of the world, except
the United States dollar, have a fixed value in terms A. Trade account in the U.S. balance of payments is
of gold. neither in a deficit nor in a surplus because of the
floating exchange rates.
C. Gold is like any other asset whose value depends
upon supply and demand. B. Capital account in the U.S. balance of payments is
neither in a deficit nor in a surplus because of the
D. Gold is the reserve asset of the International floating exchange rates.
Monetary Fund.
C. Fall in the dollar's value cannot be expected to
have any effect on the U.S. trade balance.
[Fact Pattern #2]
D. Cheaper dollar helps U.S. exporters of
One Unit of Resources Can Produce domestically produced goods.
Soybeans (tons) Chips (units)
--------------- -------------
Taiwan 6 1,500 [51] Source: CMA 1288 1-16
United States 12 1,800 One U.S. dollar is being quoted at 120 Japanese yen on
the spot market and at 123 Japanese yen on the 90-day
[47] Source: CMA 1288 1-12 forward market; hence, the annual effect in the forward
(Refers to Fact Pattern #2) market is that the
In trade between Taiwan and the United States,
A. U.S. dollar is at a premium of 10%.
A. Taiwan has an absolute advantage in producing
soybeans. B. U.S. dollar is at a premium of 2.5%.

B. The United States has a comparative advantage in C. U.S. dollar is at a discount of 10%.
producing soybeans.
D. U.S. dollar is at a premium of 0.025%.
C. Taiwan has a comparative advantage in producing
soybeans.
[52] Source: CMA 1288 1-17
D. The United States has a comparative advantage in Caroline Brown, the product manager for a U.S. computer
producing chips. manufacturer, is being asked to quote prices of desktop
computers to be used in Kuwait. The Kuwaiti government
wants the price in British pounds, for delivery next year.
[48] Source: CMA 1288 1-13 Brown knows that the general price level in the United
(Refers to Fact Pattern #2) States will increase by 3%. Her banker forecasts that the
If there were free trade between the two countries, which British pound will depreciate about 5% this year with
one of the following statements would be true? respect to the U.S. dollar. If Brown is able to quote 700
pounds for immediate delivery, the price that should be
A. Only the United States will gain from free trade. quoted for delivery to Kuwait next year is about

B. The United States would specialize in the A. 735 pounds.


production of both chips and soybeans.

6
B. 721 pounds. Interest payments on foreign loans 3,700
Gifts received from abroad 1,240
C. 757 pounds. When calculating the balance of trade for this economy, the

D. 745 pounds. A. Current account has a surplus of Pta 7,000.

B. Capital account has a surplus of Pta 4,000.


[53] Source: CMA 1288 1-18
Consider a world consisting of only two countries, Canada C. Capital account has a deficit of Pta 7,700.
and Italy. Inflation in Canada in 1 year was 5%, and in Italy
10%. Which one of the following statements about the D. Current account has a deficit of Pta 7,000.
Canadian exchange rate (rounded) during that year will be
true?
[57] Source: CMA 0691 1-16
A. Inflation has no effect on the exchange rates. Many domestic industries, such as cars and textiles, are
partially protected from foreign competition by a system of
B. The Canadian dollar will appreciate by 5%. import tariffs and import quotas. A major effect of such
tariffs and quotas is to
C. The Canadian dollar will depreciate by 5%.
A. Raise the domestic price of cars and textiles.
D. The Canadian dollar will depreciate by 15%.
B. Lower the domestic price of cars and textiles.

[54] Source: CMA 1288 1-19 C. Increase the volume of international trade in cars
When analyzing a country's balance of payments accounts, and textiles.
the
D. Reduce employment in the car and textile
A. "Current account" refers only to merchandise industries in the short run.
exports and imports.

B. "Current account" and "trade balance" are the [58] Source: CMA 0691 1-18
same. Which one of the following items represents a credit in the
U.S. balance of payment accounts?
C. "Capital account" refers to the transactions related
to the international movement of financial capital. A. U.S. imports of crude oil.

D. Country will be in financial jeopardy unless each B. Expenditures of American tourists abroad.
component in the balance of payments accounts
balances at the end of the year. C. Earnings belonging to foreign businesses that have
U.S. plants.

[55] Source: CMA 1288 1-20 D. Loans to Americans by foreigners.


Which one of the following statements about special
drawing rights (SDRs) is correct?
[59] Source: CMA 1293 1-25
A. Gold is used to determine the value of one SDR. In relation to the balance of trade, all international
transactions involving the purchase or sale of physical
B. All of the major currencies of the world, except products between domestic and foreign countries are
the U.S. dollar, have a fixed value in terms of SDRs. reflected in

C. SDRs were first introduced in 1969 by the A. The balance of the capital account.
International Monetary Fund to supplement existing
reserves. B. Official reserves held by the central banks.

D. SDRs are the principal reserve asset of the U.S. C. The official financing account.
government.
D. The trade balance in the current account.

[56] Source: CMA 1288 1-21


The following transactions were noted for an economy [60] Source: CMA 1293 1-26
whose currency is denominated in pesetas (Pta). Which one of the following did not contribute to the high
value of the U.S. dollar during the 1980s?
Amount in
Pesetas A. Relatively high, real interest rates.
---------
Imports of goods 20,300 B. A large demand for U.S. dollars.
Exports of goods 15,760
Domestic long-term investment in C. U.S. demand for foreign goods.
foreign countries 6,300
Investment by foreigners in the country 1,400 D. A stable U.S. government and currency.

7
against the dollar over this period. Comparing the returns of
the two companies, the United States company's return will
[61] Source: CMA 1293 1-27 be
The economic term used to describe the situation in which
each nation specializes in the production of goods that it A. Lower.
produces relatively more efficiently than other nations and
imports those goods that are produced relatively more B. Higher.
efficiently by other nations is called
C. The same.
A. Balance of trade.
D. Higher in the short-run but lower in the long-run.
B. Diminishing returns.

C. Relative competition. [66] Source: CMA 0694 1-4


If the central bank of a country raises interest rates sharply,
D. Comparative advantage. the country's currency will likely

A. Increase in relative value.


[62] Source: CIA 1193 IV-68
Which of the following provides the best justification for B. Remain unchanged in value.
reducing trade barriers among nations?

A. The military self-sufficiency argument. C. Decrease in relative value.

B. Diversification for stability argument. D. Decrease sharply in value at first and then return
to its initial value.
C. The infant industry argument.

D. Increased total world output argument. [67] Source: CMA 0694 1-5
All of the following are true about international trade except
that
[63] Source: CIA 0594 IV-58
Which of the following statements does not properly A. The gains from international trade depend on
describe a Eurodollar Deposit? specialization with comparative advantage.

A. Eurodollar Deposits are U.S. dollar deposits in B. Absolute advantage without comparative
banks outside of the U.S. advantage does not result in gains from international
trade.
B. Eurodollar Deposits are outside the direct control
of the U.S. monetary authorities. C. Absolute advantage is defined as the ability of one
nation to produce a product at a relatively lower
C. Eurodollar Deposit rates tend to be lower than opportunity cost than another nation.
domestic U.S. rates on equivalent instruments.
D. If there is reciprocal absolute advantage between
D. Interest rates on Eurodollar Deposits are tied to two countries, specialization will make it possible to
the London Interbank Offer Rate (LIBOR). produce more of each product.

[64] Source: CIA 0592 IV-70


A short-term speculative rise in the world-wide value of [68] Source: CMA 0695 1-23
domestic currency could be moderated by a central bank Which one of the following groups would be the primary
decision to beneficiary of a tariff?

A. Sell domestic currency in the foreign exchange A. Domestic producers of export goods.
market.
B. Domestic producers of goods protected by the
B. Buy domestic currency in the foreign exchange tariff.
market.
C. Domestic consumers of goods protected by the
tariff.
C. Sell foreign currency in the foreign exchange
market. D. Foreign producers of goods protected by the
tariff.
D. Increase domestic interest rates.

[69] Source: CMA 0695 1-24


[65] Source: CIA 1190 IV-58 Assuming exchange rates are allowed to fluctuate freely,
A U.S. company and a German company purchased the which one of the following factors would likely cause a
same stock on the German stock exchange and held the nation's currency to appreciate on the foreign exchange
stock for 1 year. The value of the German mark weakened market?

8
List A List B
A. A relatively rapid rate of growth in income that --------------- ---------------
stimulates imports. A.

B. A high rate of inflation relative to other countries. Rise Remain constant


B.
C. A slower rate of growth in income than in other
countries, which causes imports to lag behind Fall Depreciate
exports. C.

D. Domestic real interest rates that are lower than Rise Depreciate
real interest rates abroad. D.

Remain constant Appreciate


[70] Source: CIA 0595 IV-59
Interest rates received by depositors on Eurodollar
deposits tend to be higher than domestic U.S. rates on [74] Source: CIA 1196 IV-64
equivalent instruments because A company has a foreign-currency-denominated trade
payable, due in 60 days. In order to eliminate the foreign
A. Borrowers pay higher rates than domestic U.S. currency exchange-rate risk associated with the payable,
rates on equivalent instruments. the company could

B. The deposits involve different currencies. A. Sell foreign currency forward today.

C. Eurodollar deposits are for smaller amounts. B. Wait 60 days and pay the invoice by purchasing
foreign currency in the spot market at that time.
D. The Eurodollar market is outside the direct control
of the U.S. monetary authorities and has lower costs. C. Buy foreign currency forward today.

D. Borrow foreign currency today, convert it to


[71] Source: CIA 1195 IV-66 domestic currency on the spot market, and invest the
Of the following, a characteristic of Eurobonds is that they funds in a domestic bank deposit until the invoice
are payment date.

A. Always denominated in Eurodollars.


[75] Source: CIA 1196 IV-73
B. Always sold in some country other than the one in If the exchange rate has changed from 1 U.S. dollar being
whose currency the bond is denominated. worth 5 French francs to a rate of 1 U.S. dollar being
worth 5.5 French francs,
C. Sold outside the country of the borrower but are
denominated in the currency of the country in which A. The U.S. dollar has appreciated by 10%.
the issue is sold.
B. The U.S. dollar has depreciated by 10%.
D. Generally issued as registered bonds.
C. The French franc has appreciated by 20%.

[72] Source: CIA 0596 IV-69 D. The French franc has depreciated by 20%.
If a country has a freely floating exchange rate system and
is experiencing an appreciation in the external value of its
[76] Source: CIA 1196 IV-74
currency, it has Assuming that the real rate of interest is the same in both
countries, if country A has a higher nominal interest rate
A. A current account deficit and a capital account than country B, the currency of country A will likely be
surplus. selling at a

B. A current account surplus and a capital account A. Forward discount relative to the currency of
deficit. country B.

C. Shrinking official reserves. B. Forward premium relative to the currency of


country B.
D. No balance of payments surplus or deficit after
short-run exchange-rate adjustments are complete. C. Spot discount relative to the currency of country
B.

[73] Source: CIA 1195 IV-67 D. Spot premium relative to the currency of country
Two countries have flexible exchange rate systems and an B.
active trading relationship. If incomes [List A] in country 1,
everything else being equal, then the currency of country 1
will tend to [List B] relative to the currency of country 2. [77] Source: CIA 1195 IV-68
A direct effect of imposing a protective tariff on an item for

9
which there are both foreign and domestic producers is that
domestic producers will sell <List A> of the item while C. Increase by about 3%
domestic consumers consume <List B> of the item.
D. Increase by about 17%.
List A List B
------ ------
A. [82] Source: Publisher
Holland produces 90 million cases of soda and 20 million
More More pounds of cheese. To increase production of cheese to 30
B. million pounds, it must sacrifice 30 million cases of soda.
Iceland produces 65 million cases of soda and 65 million
More Less pounds of cheese, but to produce 75 million pounds of
C. cheese, it must sacrifice 10 million cases of soda. It can be
concluded that
Less More
D. A. Holland has an absolute advantage in both soda
and cheese production.
Less Less
B. Iceland has an absolute advantage in soda
production, and Holland has absolute advantage in
[78] Source: CIA 0595 IV-64 cheese production.
Which of the following measures create the most restrictive
barrier to exporting to a country? C. Iceland has an absolute advantage in both soda
and cheese production.
A. Tariffs.
D. Holland has a comparative advantage in soda
B. Quotas. production, and Iceland has an absolute advantage
with respect to cheese production.
C. Embargoes.

D. Exchange controls. [83] Source: Publisher


A U.S. company invested $100,000 in Canada for one
year at 10%. The Canadian dollar was selling at a spot rate
[79] Source: CIA 0594 IV-64 of $.65 when the investment was made and $.70 when the
Which of the following is a tariff? investment matured. What was the approximate yield on
this investment?
A. Licensing requirements.
A. 2.14%
B. Consumption taxes on imported goods.
B. 7.69%
C. Unreasonable standards pertaining to product
quality and safety. C. 10.00%

D. Domestic content rules. D. 18.46%

[80] Source: CIA 0594 IV-65 [84] Source: CIA 1196 IV-78
Which of the following is a direct effect of imposing a (Refer to Figure 3.) The graph depicts the domestic supply
protective tariff on an imported product? of and demand for a product that is also sold in the
domestic market by foreign producers. The domestic
A. Lower domestic prices on the imported item. producers are protected by a tariff of the amount Pt minus
Pw. Pt is the domestic price including the tariff, and Pw is
B. Lower domestic consumption of the item. the world price for the product. The effect of the tariff is to

C. Reduced domestic production of the item. A. Reduce the domestic price from OPw to OPt.

D. Higher sales revenues for foreign producers of the B. Reduce foreign sales in the domestic market from
item. ac to bc.

C. Increase domestic production from Ob to Oc.


[81] Source: Publisher
If the annual U.S. inflation rate is expected to be 3%, and D. Increase domestic production from Oa to Ob.
the German mark is expected to depreciate against the
U.S. dollar by 12%, a German firm importing from its U.S.
parent can expect the costs of imports denominated in [Fact Pattern #3]
marks to
U.S. production possibilities table
A. Decrease by about 12%. -----------------------------------
Production Alternatives
B. Decrease by about 5%. -----------------------

10
Product A B C D E F --------------------------------------
-- -- -- -- -- -- Production Alternatives
Lard 0 4 8 12 16 20 -------------------------------
Beef sides 40 32 24 16 8 0 Product A B C D E
Canada production possibilities table ----- ----- ----- ----- ---
------------------------------------- Cars 4,000 3,000 2,000 1,000 0
Production Alternatives Tractors 0 200 400 600 800
-----------------------
Product A B C D E F [88] Source: Publisher
-- -- -- -- -- -- (Refers to Fact Pattern #4)
Lard 0 3 6 9 12 15 In Bulgaria, the comparative cost of
Beef sides 60 48 36 24 12 0
A. 1 car is 3 tractors.
[85] Source: Publisher
(Refers to Fact Pattern #3) B. 1 tractor is 1/3 car.
If the U.S. and Canada engage in trade, the terms of trade
will be C. 1 car is 1/3 tractor.

A. Between 2 and 4 beef sides for 1 unit of lard. D. 3 tractors is 1 car.

B. Between 1/3 and 1/2 units of lard for 1 beef side.


[89] Source: Publisher
C. Between 3 and 4 units of lard for 1 beef side. (Refers to Fact Pattern #4)
Which of the following statements is not true?
D. Between 2 and 4 units of lard for 1 beef side.
A. Bulgaria should specialize in the production of
tractors.
[86] Source: Publisher
(Refers to Fact Pattern #3) B. Bulgaria has a comparative advantage in the
Assume that, prior to specialization and trade, the U.S. and production of tractors.
Canada both choose production possibility C. If each
specializes according to its comparative advantage, the C. Andorra should specialize in the production of
resulting gains from specialization and trade will be tractors.

A. 6 units of lard. D. Andorra has a comparative advantage in the


production of cars.
B. 8 units of lard.

C. 6 units of lard and 8 beef sides. [90] Source: Publisher


(Refers to Fact Pattern #4)
D. 8 units of lard and 6 beef sides. The terms of trade will be

[87] Source: Publisher A. Greater than 7 cars for 1 tractor.


(Refers to Fact Pattern #3)
Each nation produces only one product in accordance with B. Between 7 cars for 1 tractor and 5 cars for 1
its comparative advantage, and the terms of trade are set at tractor.
3 beef sides for 1 unit of lard. In this case, the U.S. can
obtain a maximum combination of 8 units of lard and C. Between 5 cars for 1 tractor and 3 cars for 1
tractor.
A. 12 beef sides.
D. Less than 3 cars for 1 tractor.
B. 24 beef sides.

C. 36 beef sides. [91] Source: Publisher


(Refers to Fact Pattern #4)
D. 48 beef sides. Assume that, if Bulgaria does not specialize, it will produce
alternative C and that, if Andorra does not specialize, it will
select alternative B. The gains from specialization will be
[Fact Pattern #4]
A. 100 cars and 100 tractors.
Bulgaria production possibilities table
---------------------------------------
Production Alternatives B. 200 cars and 200 tractors.
--------------------------------
Product A B C D E F C. 400 cars and 500 tractors.
----- ----- --- --- --- ---
Cars 1,500 1,200 900 600 300 0 D. 500 cars and 500 tractors.
Tractors 0 100 200 300 400 500
Andorra production possibilities table

11
[92] Source: Publisher
If the U.S. dollar-peso exchange rate is $1 for 9 pesos, a
product priced at 45 pesos will cost a U.S. consumer

A. $0.20

B. $5

C. $45

D. $405

[93] Source: Publisher


If a U.S. firm can buy 20,000 for $100,000, the rate of
exchange for the pound is

A. $.20

B. $5

C. $20

D. $50

[94] Source: Publisher


The spot rate of the French franc is $.90. If the spot rate
one year from now is $.85, the franc will have

A. Appreciated by 5.56%.

B. Depreciated by 5.56%.

C. Appreciated by 5.88%.

D. Depreciated by 5.88%.

12
should expedite collections and defer payments.
CMA PART 1 B
International Business Environment Answer (C) is incorrect because the U.S. company
should expedite collections and defer payments.
ANSWERS
Answer (D) is correct. The proper action would be
to increase collections and decrease payments.
Collections should be made quickly and converted
[1] Source: CMA 0676 1-34 into dollars to sustain the increase in their value as the
dollar appreciates. Decreasing payments would be
Answer (A) is incorrect because increasing profitable because, as the company exchanges dollars
productivity will lower the price of U.S. exports for foreign currency at a later date, it will receive
which will increase the amount of exports demanded, more of the foreign currency, thus lowering its real
and thus reduce the balance of payments deficit. cost.

Answer (B) is incorrect because as the rate of


inflation slows down, prices charged to overseas [4] Source: CMA 1281 1-12
buyers are reduced, which will increase exports (and
reduce the balance of payments deficit). Answer (A) is incorrect because a country will import
capital-intensive goods and export labor-intensive
Answer (C) is incorrect because if more money were goods only if it has relatively abundant labor.
given to countries the U.S. trades with, they would
buy more U.S. exports, thus correcting a balance of Answer (B) is incorrect because trade could increase
payments deficit. the welfare of the country.

Answer (D) is correct. A balance of payments deficit Answer (C) is correct. A country that has relatively
exists when the fixed or managed exchange rate is abundant capital will have a comparative advantage in
too high. "Too high" is when the fixed price is higher capital-intensive goods. Accordingly, these goods
than the equilibrium price would be if market forces should be exported. If the country has less abundant
were at work. To correct a balance of payments labor, it should import labor-intensive goods because
deficit the price of dollars must decrease or other it will be a less efficient producer of such.
means must be undertaken to raise the real value to
the fixed level. If the value of U.S. currency is Answer (D) is incorrect because an embargo on the
increased, the deficit will grow. export of capital-intensive goods is economically

irrational if the country has a comparative advantage


[2] Source: CMA 0680 1-17 in their production.

Answer (A) is incorrect because the value of the


dollar is not formally tied to gold. While there may be [5] Source: CMA 1281 1-16
a long-term relationship between gold and the value
of the dollar, there are often inverse (or random) Answer (A) is incorrect because it excludes capital
short-term fluctuations. inflows and outflows, grants, and remittances.

Answer (B) is incorrect because an exchange rate set Answer (B) is incorrect because it excludes capital
by the government is called a fixed exchange rate. inflows and outflows, grants, and remittances.
The old international monetary system which used
fixed exchange rates collapsed because of its Answer (C) is correct. The balance of payments is
inefficient handling of currency prices. defined as the excess of imports, private capital
outflows, grants, and remittances over exports and
Answer (C) is incorrect because the International private capital inflows. When there is a surplus in the
Monetary Fund has little effect on the valuation of balance of payments, more domestic goods may have
currencies. been sold abroad than were imported, and/or
foreigners may have invested more capital in the
Answer (D) is correct. Exchange rates are domestic country than domestic citizens invested
determined by the forces of supply and demand on abroad. For this reason, a surplus is considered a
the exchange markets. Often other forces try to favorable balance of payments. Just the opposite is
intervene in this process of exchange rate true for a deficit in the balance of payments.
determination, but these reflect only short-run
policies. An example of this type of policy would be Answer (D) is incorrect because it excludes exports
government or central bank intervention in the completely.
international money markets.

[6] Source: CMA 0682 1-12


[3] Source: CMA 0680 1-20
Answer (A) is incorrect because it increases the
Answer (A) is incorrect because the U.S. company demand for U.S. currency.
should expedite collections and defer payments.
Answer (B) is incorrect because it represents an
Answer (B) is incorrect because the U.S. company increase in demand for U.S. assets by foreigners.

13
not operate as a subsidy to exports.
Answer (C) is incorrect because it reflects an
increase in U.S. goods and services demanded by
foreigners. [10] Source: CMA 1282 1-14

Answer (D) is correct. Since debits represent Answer (A) is correct. Given a spot rate value of
unfavorable items, expenditures by U.S. residents $1.8655 and a forward rate of $1.8723, the value of
vacationing abroad require debits. Buying French the pound is greater in the futures market than in the
goods and services means using dollars to acquire spot market. Accordingly, the price is being quoted
francs, thus increasing the demand for French at a premium because speculators expect the
currency. currency to appreciate.

Answer (B) is incorrect because the price is quoted


[7] Source: CMA 0682 1-13 at a discount if the spot rate is higher than the forward
rate.
Answer (A) is incorrect because it reflects an
increase in demand for foreign currency by Answer (C) is incorrect because undervaluation
refers to comparisons of currencies.
Americans.
Answer (D) is incorrect because overvaluation refers
Answer (B) is incorrect because it is a private capital to comparisons of currencies.
outflow and would appear as a debit in the balance of
payments account.
[11] Source: CMA 1282 1-17
Answer (C) is incorrect because it represents imports
by the U.S. of insurance services. Answer (A) is incorrect because scarcity causes a
higher opportunity cost.
Answer (D) is correct. Exports help a country's
balance of payments and are therefore considered a Answer (B) is correct. If one good requires
credit in the U.S. balance of payments account. comparatively large amounts of a resource, a country
Therefore, the Iowa farmer's export of grain to with a relatively large amount of that resource should
Turkey would be recorded as a credit. have the advantage in the production of that good.

Answer (C) is incorrect because the country may


[8] Source: CMA 1282 1-12 have relatively less abundant natural resources.

Answer (A) is incorrect because a clean float system Answer (D) is incorrect because the country may
does not exist owing to central bank intervention. have relatively less abundant capital.

Answer (B) is correct. Today's international


monetary system usually permits exchange rates to [12] Source: CMA 1282 1-18
float freely. However, central banks occasionally
intervene to avoid large fluctuations. Accordingly, the Answer (A) is incorrect because a quota is a
system is called a managed or dirty float system. restriction on a quantity of units.

Answer (C) is incorrect because, as a result of the Answer (B) is incorrect because a tariff is a per unit
floating exchange rate system, rates are not stable. tax.

Answer (D) is incorrect because the gold-based Answer (C) is incorrect because a quota is a
system was abandoned in 1973. restriction on quantity, not a subsidy.

Answer (D) is correct. A tariff is a duty imposed on


[9] Source: CMA 1282 1-13 imports. A quota is a restriction on the quantity of a
good which may be imported.
Answer (A) is correct. If a currency is overvalued
relative to a foreign currency, it will purchase more of
that currency, thereby encouraging imports of foreign [13] Source: CMA 1282 1-19
goods. Similarly, exports are discouraged because
the foreign currency will purchase less of the Answer (A) is incorrect because a unilateral import
overvalued currency. Therefore, an overvalued quota is imposed by the importing country on the
currency operates as a tax on exports and a subsidy exporting country without agreement with the other
to imports. country.

Answer (B) is incorrect because it describes the Answer (B) is incorrect because, if the quota is
effect of an undervalued currency. adhered to, it will be quite meaningful.

Answer (C) is incorrect because overvaluation does Answer (C) is correct. A voluntary export quota is in
not operate as a tax on imports. reality a form of negotiated import quota from the
perspective of the other country involved.
Answer (D) is incorrect because overvaluation does

14
Answer (D) is incorrect because of the voluntary increase manufacturing costs; they only permit those
nature of the quota. with higher costs to compete effectively.

Answer (C) is incorrect because the benefit of the


[14] Source: CMA 1285 1-25 subsidy accrues to the inefficient domestic producer.

Answer (A) is incorrect because it concerns capital Answer (D) is correct. Trade restrictions are
movements, not trade. Also, excessive capital designed to protect domestic industries that cannot
outflows produce a negative effect on the balance of effectively meet foreign competition. Tariffs and
payments. quotas therefore cause consumers to pay higher
prices and to consume fewer goods and services. In
Answer (B) is correct. When a nation engages in effect, consumers pay a subsidy to domestic
international trade, it mostly trades goods and producers. The long-term results are a reduction in
services in which it has a comparative advantage for trade and misallocation of resources to less efficient
goods and services of which it is a relatively less industries.
efficient producer. A nation may thus concentrate on
production of those goods for which it has a
comparative advantage. Consequently, imported [17] Source: CMA 1285 1-28
products are sold for real prices lower than if they
were produced in the importing nation. The result is Answer (A) is incorrect because foreign investment
greater availability of goods and services in the may be welcomed under the appropriate conditions.
domestic market.
Answer (B) is incorrect because the EU provides
Answer (C) is incorrect because import duties are incentives to trade with other members, not
charged to domestic consumers and thus result in no nonmembers.
increase in national resources.
Answer (C) is incorrect because currencies are not
Answer (D) is incorrect because the export industries affected by bloc membership.
may well account for only a small fraction of a
nation's activity, so the greater availability of goods Answer (D) is correct. A trading bloc provides
and services is a more significant effect of trading incentives to member nations and
international trade than the benefits to exporters. discriminates against nonmember nations. For
example, the European Union calls for abolition of
internal tariffs and import quotas, free movement of
[15] Source: CMA 1285 1-26 capital and labor within the market, and
implementation of common policies for the member
Answer (A) is incorrect because the passing of "buy nations. However, the EU also imposed a common
American" laws could result in a decline in overall system of tariffs on goods of nonmember nations.
domestic consumption, higher prices, and retaliatory
foreign action.
[18] Source: CMA 1285 1-30
Answer (B) is incorrect because a country does not
benefit from restricting its exports. Answer (A) is correct. Currency devaluations result
in a change in the balance of payments. A devaluation
Answer (C) is correct. Dumping is the practice of means that other currencies will buy more of the
supporting exports by selling products at a lower devaluing nation's currency, and the prices of goods
price in foreign markets than in the domestic market. denominated in the devalued currency are therefore
The result is that foreign goods (such as certain items cheaper. A devaluation usually results in an increase
produced in the Far East) can be purchased in the in exports, a decrease in imports (caused by higher
U.S. at a price much lower than would be charged by relative input prices), and an improved balance of
a U.S. manufacturer. Since dumping lowers the price trade.
of foreign goods, the appropriate remedy would be
for the importing nation to impose a tariff that would Answer (B) is incorrect because a devaluation will
reduce the price differential. encourage exports.

Answer (D) is incorrect because denying "most Answer (C) is incorrect because a devaluation will
favored nation" treatment would make trade with a discourage imports and may encourage domestic
country more difficult and would be a more extreme inflation. Foreign goods will be more expensive.
remedy than necessary.
Answer (D) is incorrect because devaluation most
likely occurs when a country is losing its reserves.
[16] Source: CMA 1285 1-27

Answer (A) is incorrect because the government is [19] Source: CMA 1285 1-31
not trying to equitably distribute income. Import
restrictions are either politically expedient, or Answer (A) is incorrect because foreign investments
intended to support critical industries that might be in the United States is a factor in the balance of
needed in time of war. payments but not trade.

Answer (B) is incorrect because restrictions do not Answer (B) is incorrect because U.S. investments in

15
foreign countries is a factor in the balance of should buy pounds now. If the dollar depreciates
payments but not trade. against the pound in the next 90 days, the gain on the
forward exchange contract would offset the loss from
Answer (C) is incorrect because exports increase the having to pay more dollars to satisfy the liability.
balance of trade.
Answer (B) is incorrect because selling pounds
Answer (D) is correct. The balance of payments would compound the risk of loss for someone who
embraces all payments made by one nation to has incurred a liability. However, it would be an
another, including capital movements. The balance of appropriate hedge of a receivable denominated in
trade is the difference between imports and exports pounds.
of goods and services over a given period. In
T-account form, exports are credits and imports Answer (C) is incorrect because the importer needs
debits. Assuming that a credit balance reflects a pounds, not dollars.
positive balance of trade, imports will decrease a
positive balance while exports will increase it. Answer (D) is incorrect because, although buying
pounds might be equivalent to selling dollars for
pounds, this is not the best answer. This choice does
[20] Source: CMA 1285 1-32 not state what is received for the dollars.

Answer (A) is incorrect because a recession would


result in less U.S. demand for raw materials from [23] Source: CMA 1285 1-35
abroad and a reduction in funds available to the
underdeveloped nations to pay debts. Answer (A) is incorrect because the World Trade
Organization (WTO) is an attempt by the signatory
Answer (B) is correct. An expanding U.S. economy nations to reduce tariffs and import quotas.
would result in greater demand for raw materials from
these countries. Also, since the money supply and Answer (B) is incorrect because the World Bank
interest rates are inversely proportional (when the provides credits for development purposes.
money supply is rising, interest rates are falling), less
developed nations could borrow again at lower rates. Answer (C) is incorrect because the U.S. Food for
Moreover, if the money supply is rising, inflation Peace program permits other countries to purchase
might increase and U.S. dollars would become U.S. farm products with other currencies.
cheaper, thereby easing the burden of foreign debtors
with obligations payable in dollars. Answer (D) is correct. The resources of the IMF
consist of a pool of currency from which short-term
Answer (C) is incorrect because an expansion of loans can be made to member nations who are
lending authority could only increase the debt experiencing temporary balance of payments
outstanding and make it less possible for less problems. Special drawing rights (SDRs) have also
developed countries to service their debts. been created on the books of IMF to serve as an
additional reserve for member nations to use in
Answer (D) is incorrect because tariffs would reduce settling accounts.
exports to the U.S. and thus the funds available for
debt service.
[24] Source: CMA 0686 1-23

[21] Source: CMA 1285 1-33 Answer (A) is incorrect because efficient trade is not
meaningful in this context.
Answer (A) is incorrect because purchasing-power
parity is achieved through floating exchange rates. Answer (B) is incorrect because diminishing returns is
not meaningful in this context.
Answer (B) is incorrect because the
purchasing-power parity exchange rate is a long-run Answer (C) is incorrect because relative competition
measure, but the market rate may reflect short-term is not meaningful in this context.
or medium-term conditions.
Answer (D) is correct. The doctrine of comparative
Answer (C) is incorrect because purchasing-power advantage relates to comparative costs within one
parity does not affect the valuation of currency. country. It holds that a country should produce those
products in which it has a comparative advantage, not
Answer (D) is correct. The purchasing-power parity necessarily those products in which it has an absolute
theorem states that, in the long run, the real price of a advantage. The doctrine suggests that a country
good in country A will equal the price of the same should produce those products for which the greatest
good in country B when the prices are expressed in a efficiencies are attainable even if it could also produce
common currency and converted at the current other goods more efficiently than another nation. In
exchange rate (adjustments for tariffs, taxes, or the long run, importing a product in which a country
transportation cost may need to be made). has an absolute advantage but not a comparative
advantage will result in an overall increase in global
production.
[22] Source: CMA 1285 1-34

Answer (A) is correct. The American importer [25] Source: CMA 1286 1-16

16
Answer (C) is incorrect because 5% is the difference
Answer (A) is incorrect because the balance in the between the currency depreciation and the inflation
current account contains financial balances, but the rate.
balance of trade is concerned only with the balance
of transactions in goods and services. Answer (D) is correct. Assuming the original
exchange rate is $1 to 2,000 lira and that U.S.
Answer (B) is incorrect because the capital account inflation is 5%, the cost in lira to purchase what once
records net capital movements, not the balance of cost $1 will now be 2,100 lira (2,000 x 1.05).
trade. However, if the lira also depreciates by 10%, that is,
if the lira is expected to be worth 90% of its current
value against the dollar, the exchange rate before
Answer (C) is incorrect because the balance of all inflation will be $1 to 2,222 lira (2,000 .9). At this
international transactions includes items that are not rate, 2,333 lira (2,222 x 1.05) will be required to
goods or services, such as net capital movements, purchase $1.05. Lira costs will thus increase by over
government transactions, and remittances. 16.6% (333 2,000).

Answer (D) is correct. The balance of payments


embraces all payments made by one nation to [28] Source: CMA 1286 1-19
another, including capital movements. The balance of
trade is the difference between imports and exports Answer (A) is correct. An overvalued exchange rate
of goods and services over a given period. In is a tax on exports because they will be overvalued in
T-account form, exports are credits and imports terms of the foreign currency. For example, if the true
debits. Assuming that a credit balance reflects a value of $1 is 5 marks but the exchange rate is $1 to
positive balance of trade, imports will decrease a 6 marks, the cost of goods priced in dollars will
positive balance while exports will increase it. include a 20% tax for holders of marks. It is also a
subsidy to imports because the overly high exchange
rate causes the price of foreign goods and services to
[26] Source: CMA 1286 1-17 be undervalued. In the same example, $1 will buy 6
marks' worth of imports instead of 5, a 20% subsidy.
Answer (A) is incorrect because in 1971
convertibility of gold into dollars was ended by the Answer (B) is incorrect because devaluation taxes
U.S. government. This effectively destroyed the imports and subsidizes exports.
international system of fixed exchange rates. SDRs
serve the same function as gold once did in settling Answer (C) is incorrect because both will be
accounts between nations. Their value is based on a affected.
weighted average of currencies, not on the gold price.
Answer (D) is incorrect because both will be
Answer (B) is correct. Special drawing rights (SDRs) affected.
have been created on the books of the IMF by
agreement of the IMF members to serve as an
additional reserve for member nations to use in [29] Source: CMA 1286 1-21
settling accounts. They are based on the weighted
average value of several currencies. Answer (A) is incorrect because Japan and Germany
should specialize in making radios and TV sets,
Answer (C) is incorrect because in 1971 respectively, and trade for what they do not produce.
convertibility of gold into dollars was ended by the
U.S. government. This effectively destroyed the Answer (B) is correct. Japan has a comparative
international system of fixed exchange rates. SDRs advantage in radio production because its cost of
serve the same function as gold once did in settling producing radios is a smaller fraction of its cost of
accounts between nations. Their value is based on a producing TV sets (2/4 = .5) than is true for
weighted average of currencies, not on the gold price. Germany (its fraction is 3/5 = .6). Germany has a
comparative advantage with regard to TV set
Answer (D) is incorrect because SDRs are production because its costs of producing TV sets
bookkeeping entries on the IMF books awarded in (5/3 = 1.67) is a smaller fraction of its cost of
proportion to members' contribution quotas, not to producing radios than is true for Japan (its fraction is
borrowing rights. 4/2 = 2). Under the theory of comparative
advantage, Japan should manufacture radios,
Germany should make TV sets, and both should
[27] Source: CMA 1286 1-18 trade for what they do not produce.

Answer (A) is incorrect because the combined effect Answer (C) is incorrect because Japan and Germany
of U.S. inflation and the decline in value of the lira should specialize in making radios and TV sets,
would cause the lira costs for U.S. imports to respectively, and trade for what they do not produce.
increase.
Answer (D) is incorrect because Japan and Germany
Answer (B) is incorrect because the combined effect should specialize in making radios and TV sets,
of U.S. inflation and the decline in value of the lira respectively, and trade for what they do not produce.
would cause the lira costs for U.S. imports to
increase.
[30] Source: CMA 1286 1-22

17
Answer (A) is incorrect because 2.00 televisions is
the inverse of the correct relationship. [33] Source: CMA 1287 1-24

Answer (B) is incorrect because Japan has an Answer (A) is correct. If exchange rates are allowed
absolute advantage for both products. Hence, the to fluctuate, the value of a particular currency will be
correct relationship is 2 hours of production time for determined in accordance with the supply of and
a radio and 4 hours for a television. demand for that currency. For example, if U.S.
exports to Japan are greater than imports, the dollar
Answer (C) is incorrect because Japan has an will be in great demand; thus, the dollar will be driven
absolute advantage for both products. Hence, the up in price relative to the Japanese yen. This increase
correct relationship is 2 hours of production time for in price will discourage the Japanese from buying
a radio and 4 hours for a television. U.S. goods. The decrease in Japanese purchases will
then result, in principle, in less demand for the dollar
Answer (D) is correct. Because Japan has an and a movement back toward equilibrium in the
absolute advantage for both products, the relationship export/import ratio.
between Japanese radios and Japanese televisions
would determine the price of each. Radios require 2 Answer (B) is incorrect because tight money policies
hours of production time and televisions require 4 would not affect the balance of payments but would
hours, so the price of a radio is likely to be .50 reduce economic activity generally.
televisions.
Answer (C) is incorrect because deficits can be
overcome by less drastic and counterproductive
[31] Source: CMA 1286 1-20 methods than eliminating trade.

Answer (D) is incorrect because demand for


Answer (A) is incorrect because there is no way to particular products is as important as relative costs in
tell if the rates are over- or undervalued. creating trade deficits.

Answer (B) is incorrect because the pound is selling


at a discount, not a premium. [34] Source: CMA 1287 1-25

Answer (C) is correct. A spot rate is defined as the Answer (A) is incorrect because prices will be lower
exchange rate paid for delivery of currency "on the and greater quantities will be available with
spot," i.e., today. A forward exchange rate is the international trade.
future price of the currency. If the forward rate is
greater than the spot rate, the currency is selling at a Answer (B) is incorrect because variety will be
premium. If the forward rate is less than the spot rate, increased by international trade.
the currency is selling at a discount.
Answer (C) is correct. Under the concept of
Answer (D) is incorrect because the pound is selling comparative advantage, total world output will be
at a discount, not a premium. maximized when each nation specializes in the
products in which it has the lowest opportunity costs,
that is, a comparative advantage. When nations
[32] Source: CMA 0687 1-21 specialize in what they produce most efficiently and
then exchange with others, more is produced and
Answer (A) is incorrect because the WTO is a consumed than if each nation tries to be
worldwide agreement concerning trade barriers, not self-sufficient. Specialization of labor is beneficial for
exchange rates. Today, moreover, exchange rates are individuals; the same principle applies to nations.
not pegged (fixed) but are allowed to float.
Answer (D) is incorrect because competition will be
Answer (B) is incorrect because the IMF was increased when more producers are in the market.
founded in 1944 to stabilize exchange rates.

Answer (C) is correct. International trade agreements [35] Source: CMA 1287 1-26
provide regulatory authority for businesses in
international trade. The WTO, which was established Answer (A) is incorrect because U.S. investment
on January 1, 1995, is the product of the Uruguay abroad has proven to be beneficial.
Round of international trade negotiations. It is a
permanent body with a secretariat based in Geneva, Answer (B) is incorrect because both nations should
Switzerland The WTO Agreement is a permanent set benefit from international trade. Under the concept of
of commitments by more than 120 nations designed comparative advantage, no country will be worse off
to prohibit trade discrimination among member through international trade.
nations and between imported and domestic
products. Answer (C) is incorrect because the U.S. has other
advantages not possessed by countries with low
Answer (D) is incorrect because the WTO is a labor costs.
worldwide agreement concerning trade barriers, not
exchange rates. Today, moreover, exchange rates are Answer (D) is correct. Imports can raise the standard
not pegged (fixed) but are allowed to float. of living because more goods are available to

18
consumers. Tariffs, quotas, and other trade restraints depreciated against foreign currencies.
are undesirable because free trade will maximize the
total benefit to consumers worldwide (under the Answer (B) is incorrect because the U.S. trade
concept of comparative advantage). balance of payments should improve.

Answer (C) is correct. The decline in the value of the


[36] Source: CMA 1287 1-27 dollar reduces the prices of U.S. goods to foreigners
and should increase exports. Also, foreign goods will
Answer (A) is incorrect because trade restricts full be higher priced (in dollars) and imports from foreign
inflation since incentives to charge lower prices for countries should decrease, thus helping the U.S.
domestic products are removed. balance of payments.

Answer (B) is incorrect because domestic industries Answer (D) is incorrect because U.S. imports will
support protection from imports to reduce decline. Foreign goods will be higher priced than
competition. before.

Answer (C) is incorrect because consumption is


reduced owing to higher prices and reduced supply. [40] Source: CMA 0688 1-21

Answer (D) is correct. Tariffs lead to higher prices on Answer (A) is correct. The U.S. dollar is the most
imported products. Similarly, the imposition of quotas widely used currency in international markets today.
leads to higher prices through an artificial limitation on It is considered much more stable than any of the
supply. third-world currencies. Thus, many third-world
countries rely on the U.S. dollar for foreign trade.

[37] Source: CMA 1287 1-28 Answer (B) is incorrect because the Euro is not as
heavily used as the U.S. dollar.
Answer (A) is incorrect because the mark has
appreciated (increased in value) relative to the dollar. Answer (C) is incorrect because the Japanese yen is
not as heavily used as the U.S. dollar.
Answer (B) is correct. The dollar has declined in
value relative to the mark. If an American had Answer (D) is incorrect because the Swiss franc is
previously wished to purchase a German product that not as heavily used as the U.S. dollar.
was priced at 12 marks, the dollar price would have
been $2. After the decline in value, the dollar cost of
the item has increased to $3. Therefore, imports from [41] Source: CMA 0688 1-22
Germany should decrease and exports increase.
Answer (A) is incorrect because governments have
Answer (C) is incorrect because dollars will buy only temporary influence, if any, on the setting of
fewer German products. exchange rates.

Answer (D) is incorrect because U.S. exports should Answer (B) is incorrect because the International
increase. For example, if a U.S. product previously Monetary Fund has only temporary influence, if any,
cost $3, the price to Germans would have been 18 on the setting of exchange rates.
marks. With the decline of the dollar, the Germans
can now buy the $3 item for 12 marks. Answer (C) is correct. Although currencies can be
supported by various means for short periods, the
primary determinant of exchange rates is the supply
[38] Source: CMA 1287 1-29 of and demand for the various currencies. Under
current international agreements, exchange rates are
Answer (A) is incorrect because the demand for allowed to "float." During periods of extreme
dollars, not the supply, will be affected by the fluctuations, however, governments and control
decision to purchase additional U.S. products. banks may intervene to maintain stability in the
market.
Answer (B) is incorrect because the demand for
dollars, not the supply, will be affected by the Answer (D) is incorrect because they have only
decision to purchase additional U.S. products. temporary influence, if any, on the setting of exchange
rates.
Answer (C) is incorrect because the dollar should
appreciate relative to the yen owing to the increased
demand for dollars. [42] Source: CMA 0688 1-23

Answer (D) is correct. The increase in demand for Answer (A) is correct. An individual who purposely
U.S. products will increase the demand for the accepts exchange rate risk is a speculator.
dollars necessary to pay for those products. Speculators buy and sell foreign currencies in
anticipation of favorable changes in rates.

[39] Source: CMA 1287 1-30 Answer (B) is incorrect because an arbitrageur is
someone who simultaneously buys foreign currency in
Answer (A) is incorrect because the dollar has one market and sells in another market at a slightly

19
higher price. Thus, the arbitrageur's risk is slight. currency on the market.

Answer (C) is incorrect because hedging avoids the


risk of foreign currency transactions for those who do [45] Source: CMA 0688 1-29
not seek to gain from fluctuations in exchange rates.
Hedging is the sale or purchase of a forward Answer (A) is incorrect because this restriction
exchange contract to offset a possible exchange rate increases the price to U.S. consumers. The reduced
loss. When a forward exchange contract is intended supply may not be matched by reduced demand.
and is effective as an economic hedge against an
exposed net asset or net liability position (e.g., an Answer (B) is incorrect because the government will
outstanding receivable or liability denominated in a not directly benefit from the restriction on auto
foreign currency), any exchange gain or loss on the imports from Japan. The restriction benefits U.S.
forward contract will offset any exchange gain or loss manufacturers and helps equalize the balance of
on the exposed net asset or net liability position. payments.
Thus, no exchange gain or loss will result.
Answer (C) is correct. A voluntary agreement to
Answer (D) is incorrect because exporters and restrict exports to the United States forestalls
importers are likely to engage in hedging to avoid possible trade restrictions. Such an agreement is
exchange rate risk. more likely if Japan's domestic profit margins are
sufficiently high that its dependence on exports to the
U.S. has been reduced.
[43] Source: CMA 0688 1-24
Answer (D) is incorrect because Japan will want to
Answer (A) is incorrect because SDRs have nothing export more expensive cars to the U.S. The
to do with inflation. higher-priced cars will have higher margins of profit.

Answer (B) is correct. Special drawing rights (SDRs)


were created by the International Monetary Fund [46] Source: CMA 0688 1-30
(IMF) to enable countries to cope with temporary
foreign exchange liquidity problems. SDRs are Answer (A) is incorrect because although most
granted in proportion to the IMF quotas with the exchanges quote the price of gold in U.S. dollars, the
approval of 85% in voting power of the participants. dollar's value is not linked to that of gold.
If an IMF country has insufficient amount of a
currency needed for payment of a trade deficit, it can Answer (B) is incorrect because floating exchange
transfer SDRs instead. rates have existed since about 1973. Tying currency
values to a gold standard, in effect, fixes exchange
Answer (C) is incorrect because SDRs are created rates.
by an agreement among governments; they are entries
in the accounts of the IMF that have been decreed by Answer (C) is correct. Gold has no special role in the
participating countries to be acceptable in lieu of modern international monetary system. The present
other currencies. They are not used by commercial system is based upon managed floating currency
banks. exchange rates. Consequently, gold is treated as a
commodity, the price of which depends upon supply
Answer (D) is incorrect because any country and demand.
belonging to the IMF can use SDRs.
Answer (D) is incorrect because the only reserves of
the IMF are international currencies.
[44] Source: CMA 0688 1-25

Answer (A) is incorrect because currencies do not [47] Source: CMA 1288 1-12
have an inherent tendency to depreciate or
appreciate. Answer (A) is incorrect because Taiwan does not
have an absolute advantage with respect to either
Answer (B) is correct. Exchange rates "float" when product.
they are set by supply and demand, not by agreement
among countries. In a managed float, central banks Answer (B) is correct. Given two countries and two
buy and sell currencies at their discretion to avoid products, each country will always have a
erratic fluctuations in the foreign currency market. comparative advantage with respect to one of the
The objective of such transactions is to "manage" the products. Each country may or may not have an
level at which a particular currency sells in the open absolute advantage. In this case, the U.S. has an
market. For instance, if there is an oversupply of a absolute advantage with respect to both products
country's currency on the foreign currency market, because with one unit of resources it can produce
the central bank will purchase that currency to more soybeans (12 to 6) and more chips (1,800 to
support the market. 1,500) than Taiwan. Comparatively, the U.S. has an
advantage with respect to soybeans. Comparative
Answer (C) is incorrect because currencies no longer advantage is measured by determining which product
have to be supported by gold. can be manufactured more cheaply (for the lower
opportunity cost) in comparison to the other country.
Answer (D) is incorrect because central banks, not For 1 ton of soybeans, the U.S. opportunity cost is
private business people, manage the quantity of 150 units (1,800/12) of chips. The opportunity cost

20
for Taiwan is 250 units (1,500/6) of chips.

Answer (C) is incorrect because the U.S. has the [50] Source: CMA 1288 1-15
comparative advantage in the production of
soybeans, and Taiwan has the comparative Answer (A) is incorrect because the trade account
advantage in the production of chips. Taiwan's still has a deficit despite the cheaper dollar. Imports
opportunity cost for the production of one unit of continue to exceed exports because patterns of
chips is .004 ton (6/1,500) of soybeans. U.S. chip consumption are slow to change.
production has an opportunity cost of .0067 ton
(12/1,800) of soybeans. Answer (B) is incorrect because the capital account
benefits from the cheaper dollar. Foreigners can buy
Answer (D) is incorrect because the U.S. has the more dollars with fewer yen, marks, etc. Moreover,
comparative advantage in the production of foreign capital inflow increases because of the federal
soybeans, and Taiwan has the comparative government's budget deficits. Hence, the U.S.
advantage in the production of chips. Taiwan's experiences a net capital inflow.
opportunity cost for the production of one unit of
chips is .004 ton (6/1,500) of soybeans. U.S. chip Answer (C) is incorrect because the fall in the dollar
production has an opportunity cost of .0067 ton has a positive effect on the nation's trade deficit.
(12/1,800) of soybeans. Exports increase and imports decrease.

Answer (D) is correct. A decline in the value of the


[48] Source: CMA 1288 1-13 dollar relative to other currencies lowers the price of
U.S. goods to foreign consumers. Thus, exporters of
Answer (A) is incorrect because both countries will domestically produced goods benefit. A low value of
benefit from free trade. Specialization results in the the dollar also decreases imports by making foreign
optimal total output. goods more expensive.

Answer (B) is incorrect because the U.S. will


specialize in the production of soybeans. [51] Source: CMA 1288 1-16

Answer (C) is incorrect because Taiwan will produce Answer (A) is correct. The price difference of three
chips and export them to the U.S. yen is a difference of 2 % (3 120) for 90 days.
Annualizing this difference (multiplying by 4)
Answer (D) is correct. Countries should specialize in produces an annual premium of 10%. Because the
the production and export of products for which they 90-day price of the dollar is higher than the current
have a comparative advantage. They should import price, it is said that the dollar is at a premium on the
those products for which they do not have a 90-day forward market.
comparative advantage. Thus, Taiwan will specialize
in the production of chips, and the U.S. will specialize Answer (B) is incorrect because 2.5% is the premium
in the production of soybeans. for 90 days.

Answer (C) is incorrect because the effect is a 10%


[49] Source: CMA 1288 1-14 premium, not discount.

Answer (A) is incorrect because 1 ton of soybeans Answer (D) is incorrect because the 90-day effect is
for 250 chips is the opportunity cost for Taiwan. a 2.5% or 0.025 premium.

Answer (B) is correct. Taiwan's opportunity cost for


1 ton of soybeans is 250 units of chips. The U.S.'s [52] Source: CMA 1288 1-17
opportunity cost for 1 ton of soybeans is 150 units of
chips (1800/12). Thus, the lowest price that the U.S. Answer (A) is incorrect because 735 pounds results
should charge is 150 units of chips per ton of from considering only the effect of the 5% decline in
soybeans. Moreover, Taiwan should not pay more the pound.
than 250 units of chips for imported soybeans
because they could be made domestically for that Answer (B) is incorrect because 721 pounds results
price. Therefore, the trading price must be between from considering only the effect of the 3% U.S.
150 and 250 units of chips per ton of soybeans if inflation rate.
trade is to be advantageous.
Answer (C) is correct. Two factors are to be
Answer (C) is incorrect because the lowest price the considered: (1) the 3% inflation rate and (2) the 5%
U.S. should charge is 150 units of chips per ton of decline in the pound. Considering the inflation rate
soybeans and the maximum price that Taiwan would first, 3% should be added to the immediate delivery
be willing to pay is 250 units of chips per ton of price to arrive at a future price of 721 pounds.
soybeans. However, to allow for the foreign currency
exchange-rate risk, an additional 5% should be
Answer (D) is incorrect because the lowest price that added to the inflation-adjusted price. The bid price
the U.S. would charge is 150 units of chips per ton of should be 757 pounds [721 pounds + (5% x 721
soybeans. Additionally, Taiwan would not be willing pounds)].
to pay more than 250 units of chips per ton of
soybeans. Answer (D) is incorrect because the effects of both

21
the 3% U.S. inflation rate and the 5% decline in the were created by the International Monetary Fund
pound need to be added to the initial 700 pounds. (IMF) to enable countries to cope with temporary
foreign exchange liquidity problems. SDRs are
granted in proportion to the IMF quotas with the
approval of 85% in voting power of the participants.
[53] Source: CMA 1288 1-18 If an IMF country has insufficient amount of a
currency needed for payment of a trade deficit, it can
Answer (A) is incorrect because inflation affects transfer SDRs instead.
exchange rates by diminishing a currency's purchasing
power. Answer (D) is incorrect because SDRs are reserves
created by the IMF for use by nations in settling
Answer (B) is correct. Since Italy has experienced international accounts.
the greater inflation, its currency should depreciate in
relation to Canada's. For example, if Canada trades
100 units of a product to Italy for a preinflation price [56] Source: CMA 1288 1-21
of $100 (the domestic price in Canada), and Italy
pays with 10,000 units of an Italian product that sells Answer (A) is incorrect because the current account
domestically for 10,000 preinflation lira, the exchange has a deficit, not a surplus, of Pta 7,000.
rate without regard to inflation is 100 lira per $1
(10,000 lira/$100). Allowing for the inflation, the 100 Answer (B) is incorrect because investment by
units of the Canadian product would sell for $105. foreigners should be deducted from, not added to,
The 10,000 units of the Italian product would sell for domestic investment in foreign countries.
11,000 lira. Thus, the new exchange rate will be Furthermore, interest payments on foreign loans
104.76 lira per $1 (11,000 lira/$105), and the price affect the current, not the capital account, and the
of the Canadian dollar will increase by 4.76% capital account has a trade deficit, not a surplus.
(rounded to 5%).
Answer (C) is incorrect because investment by
Answer (C) is incorrect because the Canadian foreigners should be deducted from, not added to,
currency will appreciate relative to Italy's since domestic investment in foreign countries.
Canadian inflation was lower.
Answer (D) is correct. The balance of trade is the
Answer (D) is incorrect because the Canadian balance of imports and exports of goods. The current
currency will appreciate relative to Italy's since account also considers trade in services (none for this
Canadian inflation was lower. economy), unilateral transfers (e.g., gifts), and
investment receipts and payments. It does not include
capital transactions. Thus, investments by foreigners
in the domestic economy and investments made in
[54] Source: CMA 1288 1-19 foreign countries will not be included in the current
account. These transactions will be reflected in the
Answer (A) is incorrect because the current account capital account. The capital account will therefore
includes exports and imports of services as well as have a deficit of 4,900 (6,300 - 1,400) pesetas, since
goods. investment in foreign countries is greater than
investment by foreigners. The current account will
Answer (B) is incorrect because the trade balance have a deficit of 7,000 pesetas (15,760 - 20,300 +
concerns goods only. 1,240 - 3,700). This question assumes that the
balance of trade is synonymous with the current
Answer (C) is correct. The balance of payments account balance. But these terms are often
accounts include all international payments made by differentiated.
one nation to another, including capital movements,
imports, exports, and unilateral transfers. The net of
exports and imports is the current account balance. [57] Source: CMA 0691 1-16
The current account balance does not include capital
transactions. The capital account reflects movements Answer (A) is correct. Import tariffs and quotas
of financial capital (investments). cause the prices of goods to increase. Tariffs directly
increase the prices paid by consumers. Quotas
Answer (D) is incorrect because the various decrease supply and therefore indirectly raise the
components are never in balance. In reality, the price. At the same time, resources will shift from
important consideration is the total, and that is more relatively efficient export industries to less efficient
of a long-run than an annual problem. protected industries. Price will rise as resources are
misallocated to high-cost producers.

[55] Source: CMA 1288 1-20 Answer (B) is incorrect because prices will increase.

Answer (A) is incorrect because the gold standard Answer (C) is incorrect because the volume of trade
was abandoned by the IMF in 1973. will decline as less efficient domestic producers
remain in or enter the market. Total worldwide real
Answer (B) is incorrect because currencies have output will also decline.
floating values.
Answer (D) is incorrect because employment in the
Answer (C) is correct. Special drawing rights (SDRs) protected industries will increase in the short run, but

22
total employment may not be affected. currencies.

Answer (D) is incorrect because the stability of the


[58] Source: CMA 0691 1-18 U.S. government and its currency made the dollar a
secure store of value in the eyes of many foreigners;
Answer (A) is incorrect because imports are debited. these foreigners therefore acquired dollars to hold as
a safeguard against inflation in their own countries.
Answer (B) is incorrect because expenditures of
American tourists abroad are treated as imports and
are therefore debited. [61] Source: CMA 1293 1-27

Answer (C) is incorrect because earnings of Answer (A) is incorrect because the balance of trade
foreigners represent outflows of foreign exchange and is the difference between imports and exports of
are debited. goods.

Answer (D) is correct. Basically, exports of goods Answer (B) is incorrect because, beyond some level
and services are credited to the U.S. balance of of output, returns diminish as more and more units of
payments accounts and imports are debited. an input are added to the production process.
Similarly, capital movements may be debited or
credited. For example, transfers of capital from Answer (C) is incorrect because relative competition
foreigners to Americans, such as loans, are credits in is not a term relevant to international economics.
the accounts. In effect, these loans constitute exports
of debt instruments and increase the supply of foreign Answer (D) is correct. The relevant concept is
exchange available for the U.S. comparative advantage, which compares the costs of
inputs within a single country. In contrast, the concept
of absolute advantage compares the costs of inputs
[59] Source: CMA 1293 1-25 between countries. It is possible that a country might
have an absolute advantage with respect to every
Answer (A) is incorrect because the capital account product, but comparative advantage is different from
includes capital movements only; the direction of absolute advantage. A particular nation can have a
capital movements is influenced by the prevailing comparative advantage even though it does not have
interest rates in each nation. an absolute advantage. For example, assume that
Country A can produce Item X for $100 and Item Y
Answer (B) is incorrect because official reserves are for $200 and that Country B can produce Item X for
assets held by central banks and are not necessarily $50 and Item Y for $150. B has an absolute
related to current transactions. advantage in the production of both products;
however, B has a comparative advantage in
Answer (C) is incorrect because this is a nonsense producing Item X (50 100, or 50% of the A cost,
answer. compared with 150 200, or 75% of the A cost for
Item Y). A has a comparative advantage in producing
Answer (D) is correct. The balance of payments Item Y (200 150, or 133% of the B cost, versus
represents all international payments made by one 100 50, or 200%for Item X). A nation will benefit
nation to another, including those for imports, by exporting goods in which it has a comparative
exports, investments, unilateral transfers such as advantage and importing goods in which it does not
pensions and gifts, and capital movements. The have a comparative advantage. Total output will be
principal accounts are the current account and the maximized when each nation specializes in the
capital account. The current account includes a net products in which it has the greatest comparative
trade balance in goods, net investment receipts or advantage or the least comparative disadvantage.
payments, net receipts or payments for services, and
the balance of unilateral transfers.
[62] Source: CIA 1193 IV-68

[60] Source: CMA 1293 1-26 Answer (A) is incorrect because military
self-sufficiency is an argument for increasing trade
Answer (A) is incorrect because, in the 1980s, high barriers.
real interest rates made investments in the U.S. more
attractive. Thus, demand for U.S. dollars increased. Answer (B) is incorrect because diversification for
stability is an argument for increasing trade barriers
Answer (B) is incorrect because a large demand for by promoting industrial diversification and less
dollars drives up the price of dollars relative to other dependence on other nations for certain products.
currencies.
Answer (C) is incorrect because protection of infant
Answer (C) is correct. Many factors influence the industries is an argument for increasing trade barriers
value of a country's currency on the international for the purpose of allowing new domestic industries
market. These factors include interest rate to establish themselves.
differentials, inflation differentials, balance of trade,
balance of payments, and stability of governments. Answer (D) is correct. The general effect of free
However, a demand by Americans for more foreign trade would be to maximize world output because
goods would drive down the price of the dollar resources in each country would be deployed most
because of the resulting increased demand for foreign efficiently according to the principle of comparative

23
advantage. Comparative advantage means that a value of the mark relative to the dollar does not affect
country can produce a greater output of certain the German company's return. However, the
goods for a given level of input than other goods. weakening of the mark reduces the number of dollars
Thus, that country should specialize in and export the it will buy, and the U.S. company's return in dollars is
goods it can produce most efficiently. Total world correspondingly reduced.
output will increase in these circumstances.
Answer (B) is incorrect because the return to the
U.S. company is adversely affected by the exchange
[63] Source: CIA 0594 IV-58 rate movement.

Answer (A) is incorrect because Eurodollar Deposits Answer (C) is incorrect because the return to the
are U.S. dollar deposits in banks outside of the U.S. U.S. company is adversely affected by the exchange
rate movement.
Answer (B) is incorrect because Eurodollar Deposits
are outside the direct control of the U.S. monetary Answer (D) is incorrect because the return to the
authorities. U.S. company was directly and adversely affected by
the exchange rate movement in the short-run, but the
Answer (C) is correct. Eurodollars are U.S. dollars return to the German company was not.
on deposit in a foreign bank. These deposits are
created when a check is drawn on a dollar deposit in
a U.S. bank and then deposited in a bank outside the [66] Source: CMA 0694 1-4
U.S. This amount is then available for lending by the
foreign bank to its customers. However, the Answer (A) is correct. Exchange rates fluctuate
depositors still hold claims denominated in dollars. depending upon the demand for each country's
Because Eurodollars are outside the direct control of currency. If a country raises its interest rates, its
the U.S. monetary authorities, U.S. banking currency will appreciate. The demand for investment
regulations with respect to reserves, insurance, at the higher interest rates will shift the demand curve
interest ceilings, etc., do not apply. The absence of for the currency to the right. The reverse holds true
these costs means that Eurodollar deposit rates tend for a decrease in interest rates.
to be higher, not lower, than domestic U.S. rates on
equivalent instruments. Answer (B) is incorrect because the currency should
increase in relative value when interest rates in the
Answer (D) is incorrect because interest rates on country rise sharply. More investors will want to earn
Eurodollar Deposits are tied to the London Interbank the higher rates of interest.
Offer Rate (LIBOR).
Answer (C) is incorrect because the currency should
increase in relative value when interest rates in the
[64] Source: CIA 0592 IV-70 country rise sharply. More investors will want to earn
the higher rates of interest.
Answer (A) is correct. In the short run, a central
bank's sale of the currency increases the supply and Answer (D) is incorrect because the currency should
reduces the price of the currency. In the long run, increase in relative value when interest rates in the
given the current system of managed floating country rise sharply. More investors will want to earn
exchange rates, changes in rates should reflect the higher rates of interest.
changes in economic conditions. In other words,
exchange rates should float. But central banks are
expected to manage the float by buying and selling [67] Source: CMA 0694 1-5
currencies to counteract the disruptive effects on
rates of such temporary factors as speculation. Answer (A) is incorrect because world output will be
maximized if each country specializes in those
Answer (B) is incorrect because buying domestic products in which they have a comparative
currency in the foreign exchange market would raise advantage.
the world-wide value of the domestic currency.
Answer (B) is incorrect because comparative
Answer (C) is incorrect because selling foreign advantage, not absolute advantage, determines the
currency would raise the world-wide value of the products in which a country should specialize.
domestic currency with respect to that foreign
currency. Answer (C) is correct. The concept of comparative
advantage compares costs within a single country. It
Answer (D) is incorrect because a central bank is the ability of one nation to produce a product at a
decision to increase domestic interest rates would relatively lower opportunity cost (benefits forgone)
make the domestic currency attractive to foreign than another nation. Absolute advantage compares
investors, and raise the value of the domestic the costs of inputs between countries. One country
currency. might have an absolute advantage with respect to
every product. However, total output will be
maximized when each nation specializes in the
[65] Source: CIA 1190 IV-58 products in which it has the lowest opportunity costs,
that is, a comparative advantage.
Answer (A) is correct. The returns on the stock are
presumably paid in marks. Hence, the change in the Answer (D) is incorrect because, given a reciprocal

24
absolute advantage, a corresponding comparative
advantage will exist. Answer (C) is incorrect because Eurodollar deposits
tend to be for larger, not smaller, amounts.
Furthermore, smaller deposits tend to earn lower, not
[68] Source: CMA 0695 1-23 higher, rates than larger deposits.

Answer (A) is incorrect because domestic producers Answer (D) is correct. Eurodollars are U.S. dollars
of export goods are not benefitted. Indeed, they may deposited in banks outside the U.S. Because it is
be harmed by retaliatory tariffs. outside the direct control of the U.S. monetary
authorities, the Eurodollar market has lower costs.
Answer (B) is correct. Despite the advantages of free For example, U.S. reserve requirements and FDIC
trade, nations often levy tariffs to discourage the premium payments do not apply in this market. A
importation of certain products. A tariff is a tax on lower cost market can offer depositors higher interest
imports intended to protect a domestic producer rates.
from foreign competition. For instance, a tariff on
imported autos benefits U. S. auto manufacturers
because it is an additional cost imposed on U.S. [71] Source: CIA 1195 IV-66
consumers of such products. The disadvantages of
the tariff are that it may protect an inefficient domestic Answer (A) is incorrect because Eurobonds are not
producer and increase prices paid by domestic always denominated in Eurodollars, which are U.S.
consumers. dollars deposited outside the U.S.

Answer (C) is incorrect because domestic consumers Answer (B) is correct. Eurobonds are, by definition,
must pay higher prices for imported goods. always sold in some country other than the one in
whose currency the bond issue is denominated. Their
Answer (D) is incorrect because the foreign advantage is that they are customarily less stringently
producers will be forced to bear an additional cost. regulated than most other bonds. Hence, transaction
costs are lower.

[69] Source: CMA 0695 1-24 Answer (C) is incorrect because foreign bonds are
denominated in the currency of the country in which
Answer (A) is incorrect because an increase in they are sold.
imports drives down the value of the nation's
currency. Answer (D) is incorrect because Eurobonds are
usually issued not as registered bonds but as bearer
Answer (B) is incorrect because a high rate of bonds, so names and nationalities of the investors are
inflation devalues a nation's currency. not recorded.

Answer (C) is correct. Assuming that exchange rates


are allowed to fluctuate freely, a nation's currency will [72] Source: CIA 0596 IV-69
appreciate if the demand for it is constant or
increasing while supply is decreasing. For example, if Answer (A) is incorrect because the total balance of
the nation decreases its imports relative to exports, payments surplus or deficit, which is the sum of the
less of its currency will be used to buy foreign current and capital account balances, is what
currencies for import transactions and more of its determines changes in official reserves and pressures
currency will be demanded for export transactions. for exchange rates to change.
Thus, the supply of the nation's currency available in
foreign currency markets decreases. If the demand Answer (B) is incorrect because the total balance of
for the currency increases or does not change, the payments surplus or deficit, which is the sum of the
result is an increase in (appreciation of) the value of current and capital account balances, is what
the currency. determines changes in official reserves and pressures
for exchange rates to change.
Answer (D) is incorrect because lower interest rates
relative to those in other countries discourage foreign Answer (C) is incorrect because, given an
investment, decreases demand for the nation's appreciation of the currency, the short-run effects are
likely to be a balance of payments surplus and
currency, and reduces its value. growing official reserves.

Answer (D) is correct. In a freely floating


[70] Source: CIA 0595 IV-59 exchange-rate system, exchange rates automatically
adjust so as to eliminate balance of payments
Answer (A) is incorrect because Eurodollar surpluses or deficits. For example, if U.S. demand for
borrowers tend to pay lower, not higher, rates. country X's currency increases, the result is a U.S.
Borrowers and depositors can both receive more deficit at the existing exchange rate because demand
favorable rates because, with its lower costs, the now exceeds the supply of X's currency at that rate.
Eurodollar market can offer smaller spreads between However, the system of floating exchange rates
borrowing and lending rates. allows the change in the relative strength of the
currencies to be reflected in their exchange rate. The
Answer (B) is incorrect because U.S. dollars are on appreciation of X's currency against the U.S. dollar,
deposit in both cases. that is, the increase in the amount of U.S. dollars

25
exchangeable for a unit of X's currency, makes U.S.
products cheaper to buyers in country X. Answer (C) is incorrect because the franc is
Furthermore, X's products are more expensive to depreciating, not appreciating.
U.S. buyers. Consequently, U.S. imports will fall,
U.S. exports will rise, and the balance of payments Answer (D) is incorrect because the franc has
deficit will decrease. depreciated by 9.09%.

[73] Source: CIA 1195 IV-67 [76] Source: CIA 1196 IV-74

Answer (A) is incorrect because, if incomes in Answer (A) is correct. If the real rates of interest are
country 1 rise, the result will be a tendency for it to equal, the country with the higher nominal interest rate
devalue relative to the currency of country 2. is expected to experience a higher rate of inflation. A
higher rate of inflation is associated with a devaluing
Answer (B) is incorrect because, if incomes in currency, so the currency of the country with the
country 1 fall, consumers in country 1 will reduce higher nominal interest rate will likely be selling at a
their imports. The resulting decrease in the supply of forward discount.
currency 1 will result in a tendency for it to appreciate
relative to the currency of country 2. Answer (B) is incorrect because the currency of
country A will be selling at a forward discount.
Answer (C) is correct. If incomes in country 1 rise,
consumers in country 1 will increase their imports Answer (C) is incorrect because the spot relationship
from country 2. The resulting increase in the supply of between the two currencies cannot be determined
currency 1 will result in a tendency for it to depreciate from the information given.
relative to the currency of country 2.
Answer (D) is incorrect because the spot relationship
Answer (D) is incorrect because, if incomes in between the two currencies cannot be determined
country 1 remain constant, the currency of country 1 from the information given.
will not tend to appreciate or depreciate relative to
the currency of country 2.
[77] Source: CIA 1195 IV-68

[74] Source: CIA 1196 IV-64 Answer (A) is incorrect because domestic producers
will sell more, and domestic consumers will consume
Answer (A) is incorrect because a forward market less, as a result of the imposition of a protective tariff.
sale of foreign currency is appropriate to hedge a
receivable denominated in a foreign currency. Answer (B) is correct. Domestic producers are not
subject to the tariff and will therefore have a price
Answer (B) is incorrect because waiting to buy the advantage over their foreign competitors. However,
currency in 60 days does not eliminate the risk of an absent such competition, the domestic price of the
adverse exchange-rate movement. item will be higher. Domestic producers will sell more
at a higher price, and domestic consumers will
Answer (C) is correct. The company can arrange to consume less following the price increase.
purchase the foreign currency today rather than in 60
days by buying the currency in the forward market. Answer (C) is incorrect because domestic producers
This hedging transaction will eliminate the will sell more, and domestic consumers will consume
exchange-rate risk associated with the trade payable. less, as a result of the imposition of a protective tariff.

Answer (D) is incorrect because this strategy would Answer (D) is incorrect because domestic producers
be comparable to a future sale of the foreign currency will sell more, and domestic consumers will consume
at a rate known today, which would not provide the less, as a result of the imposition of a protective tariff.
currency needed to pay the invoice. However, the
opposite strategy would be an effective money
market hedge. If the company converted domestic [78] Source: CIA 0595 IV-64
currency to foreign currency in the spot market today
and invested in a foreign bank deposit or treasury bill, Answer (A) is incorrect because a tariff is a tax levied
it could then use the proceeds from the foreign by a foreign government against certain imported
investment to pay the invoice in 60 days. products. A firm exporting to that country must
accept lower profits, absorbing the tariff, or increase
selling prices in the foreign country to compensate.
[75] Source: CIA 1196 IV-73 The tariff reduces profitability or competitiveness in
the foreign market but does not exclude the firm from
Answer (A) is correct. If the exchange rate changes exporting to that country.
from $1 being worth 5 francs to $1 being worth 5.5
francs, the U.S. dollar has appreciated by 10% [(5.5 Answer (B) is incorrect because a quota is a limit on
- 5) 5]. the amount of goods that an importing country will
accept in certain product categories. The effect of a
Answer (B) is incorrect because the U.S. dollar has quota is to restrict the quantity the firm can export to
appreciated in value. It will now purchase more that country but not to exclude the firm from selling in
francs. that market. The effect on revenues and profitability

26
depends on market conditions in that country.
Answer (B) is incorrect because prices paid by the
German buyer will increase. It must adjust payments
Answer (C) is correct. An embargo is a total ban on in marks upward for both U. S. inflation and German
some kinds of imports. It is an extreme form of the monetary depreciation.
import quota. Embargoes have the effect of totally
excluding the exporting firm from selling in that Answer (C) is incorrect because prices would
country and are the most restrictive type of increase by 3% simply because of inflation. This
import/export law. answer disregards the effect of the currency
depreciation.
Answer (D) is incorrect because exchange controls
limit the amount of foreign exchange that can be Answer (D) is correct. Assuming the original
transacted or the exchange rate against other exchange rate is $1 to 2 marks and that U.S. inflation
currencies. These controls limit the ability of a firm is 3%, the cost in marks to purchase what once cost
selling in the country to repatriate its export earnings $1 will now be 2.06 marks (2 x 1.03). However, if
but do not exclude the firm from selling in that the mark also depreciates by 12%, that is, if the mark
market. is expected to be worth 88% of its current value, the
exchange rate (before inflation) will be $1 to 2.2727
(2 .88) marks. At this rate, 2.3409 marks (2.2727
[79] Source: CIA 0594 IV-64 x 1.03) will be required to purchase $1. Costs in
marks will therefore increase by just over 17%
Answer (A) is incorrect because licensing (.3409/2.00).
requirements limit exports, e.g., of militarily sensitive
technology.
[82] Source: Publisher
Answer (B) is correct. Tariffs are excise taxes on
imported goods imposed either to generate revenue Answer (A) is incorrect because Iceland has the
or protect domestic producers. Thus, consumption absolute advantage with respect to cheese.
taxes on imported goods are tariffs.
Answer (B) is incorrect because Holland has the

Answer (C) is incorrect because unreasonable absolute advantage in soda production.


standards pertaining to product quality and safety are
nontariff trade barriers. Answer (C) is incorrect because Holland has the
absolute advantage with respect to soda.
Answer (D) is incorrect because domestic content
rules require that a portion of an imported good be Answer (D) is correct. Comparative advantage
made in the importing country. compares costs of multiple products within a single
country. For example, Holland has a comparative
advantage in soda because it can produce soda more
[80] Source: CIA 0594 IV-65 cheaply than cheese. Absolute advantage compares
the costs of inputs between countries. In Holland, the
Answer (A) is incorrect because a protective tariff price of 10 million pounds of cheese is 30 million
can only increase the domestic price of the imported cases of soda, or a 1-to-3 ratio. In Iceland, the price
item. of 10 million pounds of cheese is 10 million cases of
soda, or a 1-to-1 ratio. Thus, Iceland has an absolute
Answer (B) is correct. A protective tariff adds to the advantage in cheese because it can produce cheese
purchase price of imported goods. If an imported
good's sales price is higher than a comparable, less more cheaply than Holland.
expensive domestic good, consumers will purchase
the domestic good. Thus, the direct effect of imposing
a protective tariff on an imported good is lower [83] Source: Publisher
domestic consumption.
Answer (A) is incorrect because it is based on a
Answer (C) is incorrect because, as the imported reversal of the beginning and ending spot rates.
item's domestic price increases, demand for domestic
goods will increase. Thus, domestic production will Answer (B) is incorrect because it is based only on
increase, not decrease. the change in spot rates--not the 10% return.

Answer (D) is incorrect because, as the imported Answer (C) is incorrect because the nominal return
item's domestic price increases, demand for the item must be adjusted for the change in spot prices.
decreases. Lower sales revenues will result.
Answer (D) is correct. The $100,000 divided by .65
resulted in $153,846.15 being invested in terms of
[81] Source: Publisher Canadian dollars. A 10% return would increase that
amount to $169,230.76 in Canadian dollars.
Answer (A) is incorrect because prices paid by the Converting $169,230.76 in Canadian dollars to U.S.
German buyer will increase. It must adjust payments dollars (by multiplying by .7) results in $118,461.53
in marks upward for both U. S. inflation and German in U.S. dollars. Dividing the $118,461.53 by the
monetary depreciation. $100,000 original investment results in a return of

27
18.46%. production without specialization.

Answer (C) is correct. If the U.S. produces 20 units


[84] Source: CIA 1196 IV-78 of lard and keeps only 8 units, 12 units are being
traded. At a 3-to-1 trade rate, the U.S. will receive
Answer (A) is incorrect because the inclusion of the 36 beef sides for its 12 units of lard.
tariff increases the domestic price.
Answer (D) is incorrect because only 36 sides will be
Answer (B) is incorrect because foreign sales in the available at a 3-to-1 ratio.
domestic market decline from ad to bc.

Answer (C) is incorrect because domestic production [88] Source: Publisher


increases from Oa to Ob.
Answer (A) is incorrect because you have to give up
Answer (D) is correct. Without the tariff, domestic only 1/3 of a tractor to get 1 car.
production is determined by the intersection of the
Pw line with the domestic supply curve at the quantity Answer (B) is incorrect because the cost of 1 tractor
Oa. Domestic production increases from Oa to Ob is 3 cars.
as a result of the introduction of the tariff. Supply
intersects the Pt line at a higher price and at a greater Answer (C) is correct. In Bulgaria, 300 cars have to
domestic quantity, Ob. be given up to get 100 tractors, or 3 cars for 1
tractor. Thus, the cost of 1 tractor is 3 cars.
Inversely, the cost of 1 car is 1/3 of a tractor.
[85] Source: Publisher
Answer (D) is incorrect because the cost of 3
Answer (A) is correct. The U.S. must give up 8 beef tractors is 9 cars.
sides for each 4 units of lard, or a ratio of 2 beef
sides to 1 unit of lard. In Canada, the same ratio is 12
beef sides for 3 units of lard, or 4 beef sides to 1 unit [89] Source: Publisher
of lard. Thus, the exchange rate will be somewhere
between 2 and 4 beef sides to 1 unit of lard. Answer (A) is incorrect because this statement is
true.
Answer (B) is incorrect because the exchange rate
for lard will be between 1/4 and 1/2 unit to 1 beef Answer (B) is incorrect because this statement is
side. true.

Answer (C) is incorrect because the exchange rate Answer (C) is correct. Andorra should not specialize
for lard will be between 1/4 and 1/2 unit to 1 beef in the production of tractors because it does not have
side. a comparative advantage in tractors. Total output is
maximized when each country specializes in the
Answer (D) is incorrect because the exchange rate products in which it has the lower opportunity cost
for lard will be between 1/4 and 1/2 unit to 1 beef (or comparative advantage). The cost of a tractor in
side. Andorra is 5 cars (1,000 200), whereas the cost of
a tractor in Bulgaria is 3 cars (300 100). Thus,
Bulgaria has the comparative advantage with tractors.
[86] Source: Publisher
Answer (D) is incorrect because this statement is
Answer (A) is correct. Canada has a comparative true.
advantage with respect to beef and could produce 60
units, enough to cover exactly the current production
in both countries (24 + 36). The U.S. could produce [90] Source: Publisher
20 units of lard, which is 6 units greater than the
current demand of 14 (8 + 6). Answer (A) is incorrect because Andorra would not
be willing to pay more than 5 cars for each tractor,
Answer (B) is incorrect because the total increase is since cars can be produced domestically for that
only 6 units. price.

Answer (C) is incorrect because there is no increase Answer (B) is incorrect because Andorra would not
in beef sides. be willing to pay more than 5 cars for each tractor,
since cars can be produced domestically for that
Answer (D) is incorrect because there is no increase price.
in beef sides.
Answer (C) is correct. The cost of a tractor in
Andorra is 5 cars (1,000 200), whereas the cost of
[87] Source: Publisher a tractor in Bulgaria is 3 cars (300 100). Thus, the
terms of trade will be somewhere between 3 cars and
Answer (A) is incorrect because there is an increment 5 cars for each tractor.
of 12 over the current production of 24.
Answer (D) is incorrect because Bulgaria, the
Answer (B) is incorrect because 24 is the current country with the comparative advantage in tractors,

28
will require at least 3 cars for each tractor. $.90. In a year, the price will drop to $.85. At $.90,
a person can purchase 100 francs for $90. Next
year, the same 100 francs will cost $85, meaning the
[91] Source: Publisher franc will be less valuable (in terms of dollars). Thus,
the value of the franc will depreciate against the
Answer (A) is correct. Under specialization, each dollar. The amount of the decline of $5, divided by
country produces those products for which it has a the original price of $90, will result in a 5.56%
comparative advantage. Thus, Bulgaria would depreciation.
produce 500 tractors, while Andorra would produce
4,000 cars. Under alternative C for Bulgaria and Answer (C) is incorrect because the franc will
alternative B for Andorra, the total production of cars depreciate rather than appreciate.
would be 3,900 (900 for Bulgaria plus 3,000 for
Andorra). Thus, the 4,000 under specialization is an Answer (D) is incorrect because it uses .85 in the
increase of 100 cars. Bulgaria would produce 200 denominator instead of .90.
tractors, while Andorra would produce 200, a total
of 400 tractors without specialization - again, an
increase of 100.

Answer (B) is incorrect because specialization will


result in production gains of only 100 units of each
product.

Answer (C) is incorrect because specialization will


result in production gains of only 100 units of each
product.

Answer (D) is incorrect because specialization will


result in production gains of only 100 units of each
product.

[92] Source: Publisher

Answer (A) is incorrect because $0.20 is based on


an inversion of the numerator and denominator in the
calculation.

Answer (B) is correct. At a 1-for-9 rate, the price in


U.S. dollars is $5, calculated by dividing 45 pesos by
9.

Answer (C) is incorrect because $45 is the price in


pesos, not dollars.

Answer (D) is incorrect because $405 is based on


multiplying 45 and 9.

[93] Source: Publisher

Answer (A) is incorrect because $.20 is the exchange


rate for the dollar, not the pound.

Answer (B) is correct. Dividing $100,000 by


20,000 produces an exchange rate of $5 to the
pound.

Answer (C) is incorrect because the exchange rate is


$5 to the pound.

Answer (D) is incorrect because the exchange rate is


$5 to the pound.

[94] Source: Publisher

Answer (A) is incorrect because the franc will


depreciate rather than appreciate.

Answer (B) is correct. Currently, the franc sells for

29

You might also like